Question:

Given below are two statements, one labeled as Assertion (A) and the other as Reason (R).
Assertion (A): In Young’s double slit experiment, the fringes produced by red light are closer compared to those produced by blue light.
Reason (R): The fringe width is directly proportional to the wavelength of light.
In the light of the above statements, choose the correct answer from the options given below:

Show Hint

In Young’s double-slit experiment: - Fringe width is given by \( \beta = \frac{\lambda D}{d} \). - Longer wavelengths (e.g., red) produce wider fringes. - Shorter wavelengths (e.g., blue) produce narrower fringes.
Updated On: Feb 6, 2025
  • Both (A) and (R) are true, but (R) is NOT the correct explanation of (A).
  • (A) is false, but (R) is true.
  • Both (A) and (R) are true, and (R) is the correct explanation of (A).
  • (A) is true, but (R) is false.
Hide Solution
collegedunia
Verified By Collegedunia

The Correct Option is B

Solution and Explanation

Step 1: Understanding the fringe width formula. The fringe width in Young’s double-slit experiment is given by: \[ \beta = \frac{\lambda D}{d}, \] where: - \( \lambda \) is the wavelength of the light, - \( D \) is the distance between slits and screen, - \( d \) is the separation between the slits. 

Step 2: Analyzing Assertion (A). Since \( \beta \propto \lambda \), red light (\(\lambda\) is larger) produces wider fringes than blue light (\(\lambda\) is smaller). Thus, Assertion (A) is incorrect because it states the opposite. 

Step 3: Analyzing Reason (R). The fringe width is indeed proportional to the wavelength, which is a correct statement. 

Since (A) is false but (R) is true, the correct choice is: \[ \boxed{\text{(2) (A) is false, but (R) is true.}} \]

Was this answer helpful?
0
0

Top Questions on Wave optics

View More Questions